Đến nội dung

Hình ảnh

Topic về Bất đẳng thức trong Tích phân

* * * * - 6 Bình chọn

  • Please log in to reply
Chủ đề này có 125 trả lời

#61
phudinhgioihan

phudinhgioihan

    PĐGH$\Leftrightarrow$TDST

  • Biên tập viên
  • 348 Bài viết

Bài 26:
Cho $\text{f(x)}$ là hàm số liên tục cùng đạo hàm của nó trên đoạn $\text{[a,b]}$ và $\text{f(a) = 0}$. Đặt $\text{M} = \max_{\text{a} \leqslant \text{x} \leqslant \text{b}}\left | \text{f(x)} \right |$.
Chứng minh rằng :
$\text{M}^{2} \leqslant (\text{b} - \text{a})\int_{\text{a}}^{\text{b}}\text{f'}^{2}(\text{x})\text{dx}$.


Lại là Cauchy-Schwarz

$$\forall x \in [a;b] \;, f^2(x) = \left( \int_a^x f'(t)dt \right)^2 $$

$$ \le \int_a^x dt \int_a^x f'^2(t)dt \le \int_a^b dt \int_a^b f'^2(t)dt$$

$$ \le (b-a) \int_a^b f'^2(t)dt$$

Do đó $\max_{x \in [a;b] } f^2(x) \le (b-a) \int_a^b f'^2(t)dt $

Phủ định của giới hạn Hình đã gửi

Đó duy sáng tạo ! Hình đã gửi


https://phudinhgioihan.wordpress.com/

#62
Ispectorgadget

Ispectorgadget

    Nothing

  • Quản lý Toán Phổ thông
  • 2946 Bài viết
Bài 27: Cho $a\in (0;1)$, chứng minh rằng $-a\ln 2\le \int_{a^2}^a \frac{dx}{\ln x} \le -a^2 \ln 2.$

►|| The aim of life is self-development. To realize one's nature perfectly - that is what each of us is here for. ™ ♫


#63
Ispectorgadget

Ispectorgadget

    Nothing

  • Quản lý Toán Phổ thông
  • 2946 Bài viết
Bài 28: Cho $f:[0,1]\to\mathbb{R}$ là hàm khả vi sao cho $f(0)=f(1)=0$ và $|f'(x)|\leq 1,\forall x\in[0,1]$. Chứng minh rằng
$$|\int_0^1 f(t)dt|<\dfrac{1}{4}.$$

Romania-District-Olympiad-2012

►|| The aim of life is self-development. To realize one's nature perfectly - that is what each of us is here for. ™ ♫


#64
phudinhgioihan

phudinhgioihan

    PĐGH$\Leftrightarrow$TDST

  • Biên tập viên
  • 348 Bài viết

Bài 28: Cho $f:[0,1]\to\mathbb{R}$ là hàm khả vi sao cho $f(0)=f(1)=0$ và $|f'(x)|\leq 1,\forall x\in[0,1]$. Chứng minh rằng
$$|\int_0^1 f(t)dt|<\dfrac{1}{4}.$$

Romania-District-Olympiad-2012


$$ \left| \int_0^1 f(x)dx \right| \le \left| \int_0^{\frac{1}{2}} f(x)dx \right| +\left| \int_{\frac{1}{2}}^1 f(x)dx \right|$$

$$\le \left| \int_0^{\frac{1}{2}} \int_0^{x} f'(t)dt dx \right| + \left|\int_{\frac{1}{2}}^1 \left(-\int_x^1 f'(t)dt \right) dx \right| $$

$$ \le \int_0^{\frac{1}{2}} \int_0^{x} |f'(t)|dt dx +\int_{\frac{1}{2}}^1 \int_x^1 |f'(t)|dtdx $$

$$ \le \int_0^{\frac{1}{2}} \int_0^{x} dtdx +\int_{\frac{1}{2}}^1 \int_x^1 dtdx $$

$$\le \int_0^{\frac{1}{2}} xdx+\int_{\frac{1}{2}}^1 (1-x)dx $$

$$ \le \dfrac{1}{4}$$

Dấu $=$ xảy ra khi $|f'(x)|=1 \;\;, \forall x \in [0;1] $ suy ra $f(x)$ là hàm bậc nhất, do đó không thể có $f(0)=f(1)=0$, vậy trường hợp đẳng thức không thể xảy ra.

Vậy $$ \left| \int_0^1 f(x)dx \right| <\dfrac{1}{4}$$

Phủ định của giới hạn Hình đã gửi

Đó duy sáng tạo ! Hình đã gửi


https://phudinhgioihan.wordpress.com/

#65
phudinhgioihan

phudinhgioihan

    PĐGH$\Leftrightarrow$TDST

  • Biên tập viên
  • 348 Bài viết
Bài 29: (Sendov-Skordev)

Cho $f$ không âm trên $[0;1]$ và thỏa $\dfrac{f(x_1)+f(x_2)}{2} \le f(\frac{x_1+x_2}{2}) \;\;, \forall x_1,x_2 \in [0;1]$

Với mọi $n \in \mathbb{N}$, chứng minh

$$\int_0^1 f(x)dx \le 3^n \int_0^1 x^n f(x)dx $$

$$\dfrac{2}{(n+1)(n+2)}\int_0^1 f(x)dx \le \int_0^1 x^n f(x)dx \le \dfrac{2}{n+2} \int_0^1 f(x)dx $$

Phủ định của giới hạn Hình đã gửi

Đó duy sáng tạo ! Hình đã gửi


https://phudinhgioihan.wordpress.com/

#66
dangnamneu

dangnamneu

    Hạ sĩ

  • Thành viên
  • 68 Bài viết

Bài 29: (Sendov-Skordev)

Cho $f$ không âm trên $[0;1]$ và thỏa $\dfrac{f(x_1)+f(x_2)}{2} \le f(\frac{x_1+x_2}{2}) \;\;, \forall x_1,x_2 \in [0;1]$

Với mọi $n \in \mathbb{N}$, chứng minh

$$\int_0^1 f(x)dx \le 3^n \int_0^1 x^n f(x)dx $$

$$\dfrac{2}{(n+1)(n+2)}\int_0^1 f(x)dx \le \int_0^1 x^n f(x)dx \le \dfrac{2}{n+2} \int_0^1 f(x)dx $$


Gợi ý: Các bạn chứng minh bất đẳng thức thứ hai trước(Chỉ dùng tích phân từng phần thui), để ý là $f$ là hàm lõm và bằng quy nạp ta có
$${3^n} \ge \frac{{\left( {n + 1} \right)\left( {n + 2} \right)}}{2},\forall n \ge 1$$.
Từ đó suy ra kết quả bài toán.

Bài viết đã được chỉnh sửa nội dung bởi dangnamneu: 02-02-2013 - 23:26

Giáo viên môn Toán tại website : http://vted.vn


#67
Ispectorgadget

Ispectorgadget

    Nothing

  • Quản lý Toán Phổ thông
  • 2946 Bài viết
Bài 30: Cho $f$ không âm, liên tục và đồng biến trên $[0;c] (c>0)$. Chứng minh rằng $$\int_0^a f(x)dx+\int_{f(0)}^bf^{-1}(y)dy \ge ab$$ Với $a\in [0;c];b\in [f(0);c]$

Bài 31: Chứng minh $$e^x-1 <\int_0^x \sqrt{e^{2t}+e^{-t}}<\sqrt{(e^x-1)(e^x-\frac{1}{2})}\;\; \forall x>0$$

Bài 32: Tìm giá trị nhỏ nhất của $$f(n)=\int_{\frac{\pi}{4}}^{\frac{3\pi}{8}}\left(\frac{\sin^{n+2}x}{\cos^n x} +\frac{\cos^{n+2}}{\sin^nx}\right )dx,n\in \mathbb{Z}^+$$

Bài 33: Cho $m\in \mathbb{N}$. Tìm giá trị nhỏ nhất của biểu thức $$f(x)=\int_1^x t^m.e^{2t}dt-2\left(\frac{x^{m+3}}{m+3}+\frac{x^{m+2}}{m+2} \right ),\, x\ge 1$$

Bài viết đã được chỉnh sửa nội dung bởi phudinhgioihan: 03-02-2013 - 12:21

►|| The aim of life is self-development. To realize one's nature perfectly - that is what each of us is here for. ™ ♫


#68
phudinhgioihan

phudinhgioihan

    PĐGH$\Leftrightarrow$TDST

  • Biên tập viên
  • 348 Bài viết

Bài 30: Cho $f$ không âm, liên tục và đồng biến trên $[0;c] (c>0)$. Chứng minh rằng $$\int_0^a f(x)dx+\int_{f(0)}^bf^{-1}(y)dy \ge ab$$ Với $a\in [0;c];b\in [f(0);c]$

Bài 31: Chứng minh $$e^x-1 <\int_0^x \sqrt{e^{2t}+e^{-t}}<\sqrt{(e^x-1)(e^x-\frac{1}{2})}\;\; \forall x>0$$

Bài 32: Tìm giá trị nhỏ nhất của $$f(n)=\int_{\frac{\pi}{4}}^{\frac{3\pi}{8}}\left(\frac{\sin^{n+2}x}{\cos^n x} +\frac{\cos^{n+2}}{\sin^nx}\right )dx,n\in \mathbb{Z}^+$$

Bài 33: Cho $m\in \mathbb{N}$. Tìm giá trị nhỏ nhất của biểu thức $$f(x)=\int_1^x t^m.e^{2t}dt-2\left(\frac{x^{m+3}}{m+3}+\frac{x^{m+2}}{m+2} \right ),\, x\ge 1$$


Bài 30: Đã chứng minh ở đây. Đây là bất đẳng thức Young

Bài 31:

Hiển nhiên $$\int_0^x \sqrt{e^{2t}+e^{-t}}> \int_0^x e^t=e^x-1$$

Đặt $$f(x)=\int_0^x \sqrt{e^{2t}+e^{-t}}-\sqrt{(e^x-1)(e^x-\frac{1}{2})} \;\;, x>0$$

$$f'(x)=\sqrt{e^2x+e^{-x}}-\dfrac{e^x}{2} \left(\sqrt{\dfrac{e^2-1}{e^x-\frac{1}{2}}} +\sqrt{\dfrac{e^x-\frac{1}{2}}{e^x-1}}\right)$$

Đặt $t=e^x >1 $

Xét bất phương trình $f'(x)<0$

$$\Leftrightarrow \sqrt{t^2+\frac{1}{t}}<\dfrac{t}{2}\left( \sqrt{t-\dfrac{1}{2(t-\frac{1}{2})}}+\sqrt{1+\dfrac{1}{2(t-1)}} \right)$$

$$\Leftrightarrow \dfrac{8}{t^3}<\dfrac{1}{t-1}+\dfrac{1}{t-\frac{1}{2}}$$

$$\Leftrightarrow 4t^4+24t-3t^3-16t^2-8>0$$

Luôn đúng $\forall t>1$

Vậy $f'(x)<0 \;\;, \forall x>0$ , suy ra $f(x)$ nghịch biến trên $(0;+\infty)$

do đó $f(x)<f(0)=0$ , vậy có đpcm.

Bài 32:

Do $x \in [\dfrac{\pi}{4}, \dfrac{3\pi}{8}] $ thì $\sin x >0 \;, \cos x >0$ nên ta có thể thác triển $f$ lên $\mathbb{R}$

Xét $f^1(n)=\int_{\frac{\pi}{4}}^{\frac{3\pi}{8}}\left(\frac{\sin^{n+2}x}{\cos^n x} +\frac{\cos^{n+2}}{\sin^nx}\right )dx \;, n \in \mathbb{R}_+$

$${f^{1}}^{'}(n)=\int_{\frac{\pi}{4}}^{\frac{3\pi}{8}} \ln \tan x \left( \frac{\sin^{n+2}x}{\cos^n x} -\frac{\cos^{n+2}}{\sin^nx}\right )dx $$

$$\forall x \in [\dfrac{\pi}{4}, \dfrac{3\pi}{8}] ,\; \ln \tan x \ge 0 \;, \sin x \ge \cos x$$

$$\Rightarrow \frac{\sin^{n+2}x}{\cos^n x} -\frac{\cos^{n+2}}{\sin^nx} \ge \cos^2 x-\cos^2 x=0$$

$$\Rightarrow {f^{1}}^{'}(n) \ge 0$$

Vậy $f^1(n)$ đồng biến trên $[0+\infty)$ , suy ra $f^1(n) \ge f^1(0)=\dfrac{\pi}{8}$

Suy ra $f(n) \ge f(0) \dfrac{\pi}{8}$

Bài 33:

$$f'(x)=x^me^{2x}-2\left(x^{m+2}+x^{m+1} \right )=x^m\left( e^{2x}-2x-2x^2 \right) $$

Lại có $e^x \ge 1+x+\dfrac{x^2}{2} \;, \forall x \ge 0$

suy ra $e^{2x} >1+2x+2x^2 \;, \forall x \ge 1$

Do đó $f'(x) >x^m>0$ , suy ra $f(x)$ đồng biến trên $[1;+\infty)$

Suy ra $f(x) \ge f(1)=-2\left( \dfrac{1}{m+2}+\dfrac{1}{m+3} \right) $

Phủ định của giới hạn Hình đã gửi

Đó duy sáng tạo ! Hình đã gửi


https://phudinhgioihan.wordpress.com/

#69
phudinhgioihan

phudinhgioihan

    PĐGH$\Leftrightarrow$TDST

  • Biên tập viên
  • 348 Bài viết
Bài 34: (mới chế :D)

Cho $[a;b] \subset \mathbb{R} $ , $f:[a;b] \to \mathbb{R} $ có đạo hàm cấp 2 liên tục trên $[a;b]$ sao cho $f(a)+f(b)=0$. Đặt $m=\min_{x \in [a;b]} f''(x) $, chứng minh

$$\int_a^b f(x)dx \le \dfrac{m(a-b)^3}{12}$$

Bài viết đã được chỉnh sửa nội dung bởi phudinhgioihan: 05-02-2013 - 06:54

Phủ định của giới hạn Hình đã gửi

Đó duy sáng tạo ! Hình đã gửi


https://phudinhgioihan.wordpress.com/

#70
phudinhgioihan

phudinhgioihan

    PĐGH$\Leftrightarrow$TDST

  • Biên tập viên
  • 348 Bài viết
Bài 35: (mới chế :D)

Tặng Ku Kiên Kháu Khỉnh

Cho $f:[0;1] \to \mathbb{R}$ có đạo hàm cấp 2 liên tục trên $[0;1]$. Đặt $M_0=\max_{[0;1]} f(x) \;, M_2=\max_{[0;1]} f''(x) \; ,m_1=\min_{[0;1]} f'(x)$

Chứng minh

$$\forall \lambda \in [0;1] \;, \int_0^1 f(x)dx \le M_0-\dfrac{m_1}{2}\left(\lambda^2+(1-\lambda)^2 \right)+\dfrac{M_2}{6}\left(\lambda^3+(1-\lambda)^3\right)$$

Phủ định của giới hạn Hình đã gửi

Đó duy sáng tạo ! Hình đã gửi


https://phudinhgioihan.wordpress.com/

#71
Ispectorgadget

Ispectorgadget

    Nothing

  • Quản lý Toán Phổ thông
  • 2946 Bài viết
Bài 36: Cho $f: [0,1]\rightarrow R$ là hàm liên tục với $f(0)=0$. Chứng minh rằng:

$$Sup_{0\leq x \leq 1}|f(x)|\leq(\int_{0}^{1}(f'(x))^2dx)^{\frac{1}{2}}$$
Spoiler

Bài viết đã được chỉnh sửa nội dung bởi Ispectorgadget: 06-02-2013 - 01:24

►|| The aim of life is self-development. To realize one's nature perfectly - that is what each of us is here for. ™ ♫


#72
dangnamneu

dangnamneu

    Hạ sĩ

  • Thành viên
  • 68 Bài viết

Bài 34: (mới chế :D)

Cho $[a;b] \subset \mathbb{R} $ , $f:[a;b] \to \mathbb{R} $ có đạo hàm cấp 2 liên tục trên $[a;b]$ sao cho $f(a)+f(b)=0$. Đặt $m=\min_{x \in [a;b]} f''(x) $, chứng minh

$$\int_a^b f(x)dx \le \dfrac{m(a-b)^3}{12}$$


Dựa vào hai đại lượng $f(a)$ và $f(b)$ ta nghĩ ngay được ý tưởng sáng tác của tác giả cho bài toán này là dựa vào Khai triển Taylor.
Dưới đây là lời giải bài toán


Giải. Khai triển Taylor ta được
$f(a) = f(x) + f'(x)\left( {a - x} \right) + \frac{{f''({x_1})}}{2}{\left( {a - x} \right)^2}$ với ${x_1} \in \left( {a,x} \right)$và
$f(b) = f(x) + f'(x)\left( {b - x} \right) + \frac{{f''({x_2})}}{2}{\left( {b - x} \right)^2}$với ${x_2} \in \left( {x,b} \right)$.
Cộng theo vế hai đẳng thức trên suy ra
$2f(x) + f'(x)\left( {a - x} \right) + f'(x)\left( {b - x} \right) + \frac{{f''({x_1})}}{2}{\left( {a - x} \right)^2} + \frac{{f''({x_2})}}{2}{\left( {b - x} \right)^2} = 0$
Lấy tích phân hai vế trên $\left[ {a,b} \right]$ ta được
$$2\int\limits_a^b {f(x)dx + \int\limits_a^b {f'(x)\left( {a - x} \right)dx} + \int\limits_a^b {f'(x)\left( {b - x} \right)dx} } + \int\limits_a^b {\left( {\frac{{f''({x_1})}}{2}{{\left( {a - x} \right)}^2} + \frac{{f''({x_2})}}{2}{{\left( {b - x} \right)}^2}} \right)dx} = 0$$
Mặt khác ta lại có
\[\int\limits_a^b {f'(x)\left( {a - x} \right)dx} = \left( {a - b} \right)f(b) + \int\limits_a^b {f(x)dx} \].
\[\int\limits_a^b {f'(x)\left( {b - x} \right)dx} = \left( {a - b} \right)f(a) + \int\limits_a^b {f(x)dx} \].
$$\frac{{f''({x_1})}}{2}{\left( {a - x} \right)^2} + \frac{{f''({x_2})}}{2}{\left( {b - x} \right)^2} \ge \frac{m}{2}\left( {{{\left( {a - x} \right)}^2} + {{\left( {b - x} \right)}^2}} \right)$$.
Nên từ ba đẳng thức trên với đẳng thức trên ta suy ra
$$4\int\limits_a^b {f(x)dx} + \frac{m}{2}\int\limits_a^b {\left( {{{\left( {a - x} \right)}^2} + {{\left( {b - x} \right)}^2}} \right)dx} \le 0 \Rightarrow 4\int\limits_a^b {f(x)dx} \le \frac{m}{2}\left( {\frac{{{{\left( {a - b} \right)}^3}}}{3} + \frac{{{{\left( {a - b} \right)}^3}}}{3}} \right)$$.

$$ \Rightarrow \int\limits_a^b {f(x)dx} \le \frac{{m{{\left( {a - b} \right)}^3}}}{{12}}$$.

Bài toán được chứng minh hoàn toàn.

...................
~~~~~~~~~~~~~~~~~~~~~~~~~~~~~~~~~~~~~~~~~~~~~~~~~~~~~~~~~~~~~~~~~~~~~~~~~~
phudinhgioihan: Thật ra mình không dùng Taylor bởi không thể tiến sâu thêm với cách xấp xỉ đa thức bá đạo này.

Với $t \in [0;1]$, khi đó $\int_a^b f(x)dx=(x-ta-(1-t)b)\bigg|_a^b -\int_a^b (x-ta-(1-t)b) f'(x)dx $

$=(x-ta-(1-t)b)\bigg|_a^b-\frac{1}{2}((x-ta-(1-t)b)^2f'(x) \bigg|_a^b+\dfrac{1}{2}\int_a^b (x-ta-(1-t)b)^2f''(x)dx$

Chọn $t=\frac{1}{2}$ dễ dàng có đpcm.

Bài viết đã được chỉnh sửa nội dung bởi phudinhgioihan: 06-02-2013 - 19:35

Giáo viên môn Toán tại website : http://vted.vn


#73
dangnamneu

dangnamneu

    Hạ sĩ

  • Thành viên
  • 68 Bài viết

Bài 36: Cho $f: [0,1]\rightarrow R$ là hàm liên tục với $f(0)=0$. Chứng minh rằng:

$$Sup_{0\leq x \leq 1}|f(x)|\leq(\int_{0}^{1}(f'(x))^2dx)^{\frac{1}{2}}$$

Spoiler




Bài này khá đơn giản, do $f(0) = 0$ nên $f(x) = \int\limits_0^x {f'(t)dt} $.. Theo bất đẳng thức Cauchy – Schwarz ta có
$\left| {f(x)} \right| \le \int\limits_0^x {\left| {f'(t)} \right|dt} \le \int\limits_0^1 {\left| {f'(t)} \right|dt} \le \sqrt {\int\limits_0^1 {{{\left( {f'(t)} \right)}^2}dt} \int\limits_0^1 {{1^2}dt} } = \sqrt {\int\limits_0^1 {{{\left( {f'(t)} \right)}^2}dt} } $.
Bài toán được chứng minh.

Giáo viên môn Toán tại website : http://vted.vn


#74
phudinhgioihan

phudinhgioihan

    PĐGH$\Leftrightarrow$TDST

  • Biên tập viên
  • 348 Bài viết
Bài 37: Cho $[a;b] \subset \mathbb{R} \;, f:[a;b] \to \mathbb{R}$ khả vi liên tục trên $[a;b]$ sao cho $f(a)=f(b)=0$. $\forall x \in [a;b]$ đặt $g(x)=\min \{|x-a|,|x-b| \} \;, $. Chứng minh

$$\int_a^b \dfrac{f^2(x)}{g^2(x)}dx\le 4\int_a^b f'^2(x)dx$$

Bài viết đã được chỉnh sửa nội dung bởi phudinhgioihan: 18-02-2013 - 20:50

Phủ định của giới hạn Hình đã gửi

Đó duy sáng tạo ! Hình đã gửi


https://phudinhgioihan.wordpress.com/

#75
dangnamneu

dangnamneu

    Hạ sĩ

  • Thành viên
  • 68 Bài viết

Bài 35: (mới chế :D)

Tặng Ku Kiên Kháu Khỉnh

Cho $f:[0;1] \to \mathbb{R}$ có đạo hàm cấp 2 liên tục trên $[0;1]$. Đặt $M_0=\max_{[0;1]} f(x) \;, M_2=\max_{[0;1]} f''(x) \; ,m_1=\min_{[0;1]} f'(x)$

Chứng minh

$$\forall \lambda \in [0;1] \;, \int_0^1 f(x)dx \le M_0-\dfrac{m_1}{2}\left(\lambda^2+(1-\lambda)^2 \right)+\dfrac{M_2}{6}\left(\lambda^3+(1-\lambda)^3\right)$$




Mình nghĩ đại lượng $ - \frac{{{m_1}}}{2}\left( {{\lambda ^2} + {{\left( {1 - \lambda } \right)}^2}} \right)$ ở vế phải có vấn đề?? Bạn xem lại được không?

............

Giáo viên môn Toán tại website : http://vted.vn


#76
dangnamneu

dangnamneu

    Hạ sĩ

  • Thành viên
  • 68 Bài viết
Bài 38:
[Đặng Thành Nam] Cho đa thức$f(x) = A{x^3} + B{x^2} + Cx + D$ thỏa mãn điều kiện $A < 0,{B^2} - 3AC \le 0$.
Chứng minh rằng

$$\int\limits_0^1 {xf(x)dx} \le \frac{1}{2}\int\limits_0^1 {f(x)dx} $$.


Hãy tổng quát bài toán khi thay đoạn $\left[ {0,1} \right]$ bởi đoạn $\left[ {a,b} \right]$.

Bài viết đã được chỉnh sửa nội dung bởi dangnamneu: 07-02-2013 - 14:56

Giáo viên môn Toán tại website : http://vted.vn


#77
Ispectorgadget

Ispectorgadget

    Nothing

  • Quản lý Toán Phổ thông
  • 2946 Bài viết
Bài 39: (Mathlinks) Chứng minh rằng

$$\int_{\pi}^{2\pi}\frac{|sin{nx}|}{x}dx >\frac{2}{\pi}(\frac{1}{1+n}+\frac{1}{2+n}+...+\frac{1}{2n})$$

Bài 40: (Mathlinks) Chứng minh rằng $$\int_{-\frac{\pi}{2}}^{\frac{\pi}{2}}\frac{\sqrt{\cos x}}{1+e^x}<\sqrt{\frac{\pi}{2}}$$

Bài 41: (Mathlinks) Chứng minh rằng$$\int_0^1 \frac{\ln(x+1)}{x^2+x+1}\le (2\ln 2-1)\frac{\pi\sqrt{3}}{29}$$

Bài 42: (Mathlinks)Cho $a,b>0, b\neq 1$. Chứng minh $$\int_1^b a^{\log_bx}>\ln b$$

Bài viết đã được chỉnh sửa nội dung bởi Ispectorgadget: 07-02-2013 - 17:37

►|| The aim of life is self-development. To realize one's nature perfectly - that is what each of us is here for. ™ ♫


#78
Ispectorgadget

Ispectorgadget

    Nothing

  • Quản lý Toán Phổ thông
  • 2946 Bài viết

Bài 34: (mới chế :D)

Cho $[a;b] \subset \mathbb{R} $ , $f:[a;b] \to \mathbb{R} $ có đạo hàm cấp 2 liên tục trên $[a;b]$ sao cho $f(a)+f(b)=0$. Đặt $m=\min_{x \in [a;b]} f''(x) $, chứng minh

$$\int_a^b f(x)dx \le \dfrac{m(a-b)^3}{12}$$

Bài này là TH riêng của quy tắc trapezoidal rule trường hợp $n=1$.

Xem thêm ở đây.
File gửi kèm  77_Trap.pdf   45.16K   388 Số lần tải

Bài viết đã được chỉnh sửa nội dung bởi Ispectorgadget: 07-02-2013 - 18:38

►|| The aim of life is self-development. To realize one's nature perfectly - that is what each of us is here for. ™ ♫


#79
Ispectorgadget

Ispectorgadget

    Nothing

  • Quản lý Toán Phổ thông
  • 2946 Bài viết

Bài 38:
[Đặng Thành Nam] Cho đa thức$f(x) = A{x^3} + B{x^2} + Cx + D$ thỏa mãn điều kiện $A < 0,{B^2} - 3AC \le 0$.
Chứng minh rằng

$$\int\limits_0^1 {xf(x)dx} \le \frac{1}{2}\int\limits_0^1 {f(x)dx} $$.


Hãy tổng quát bài toán khi thay đoạn $\left[ {0,1} \right]$ bởi đoạn $\left[ {a,b} \right]$.

Từ giả thiết ta có $f$ là hàm liên tục thỏa mãn $f(x)\ge 0$ với $x\in [0;1]$

Ta sẽ chứng minh $\int_0^1 (x-\frac{1}{2})f(x)dx \le 0$

Theo định lý giá trị trung bình tích phân

Ta có $$\int_0^1 (x-\frac{1}{2}) =0$$

Do đó tồn tại $a$ sao cho $$\int_0^1(x-\frac{1}{2})f(x)dx=f(a)\int_0^1(x-\frac{1}{2})dx=-f(a).0$$

Mà $f(a)\ge 0; \forall a\in [0;1]$, vì vậy ta có đpcm.
@@ Nghi sai quá.

Bài viết đã được chỉnh sửa nội dung bởi Ispectorgadget: 07-02-2013 - 19:00

►|| The aim of life is self-development. To realize one's nature perfectly - that is what each of us is here for. ™ ♫


#80
dangnamneu

dangnamneu

    Hạ sĩ

  • Thành viên
  • 68 Bài viết

Từ giả thiết ta có $f$ là hàm liên tục thỏa mãn $f(x)\ge 0$ với $x\in [0;1]$

Ta sẽ chứng minh $\int_0^1 (x-\frac{1}{2})f(x)dx \le 0$

Theo định lý giá trị trung bình tích phân

Ta có $$\int_0^1 (x-\frac{1}{2}) =0$$

Do đó tồn tại $a$ sao cho $$\int_0^1(x-\frac{1}{2})f(x)dx=f(a)\int_0^1(x-\frac{1}{2})dx=-f(a).0$$

Mà $f(a)\ge 0; \forall a\in [0;1]$, vì vậy ta có đpcm.
@@ Nghi sai quá.


Như này sai rùi em, điều kiện đề bài cho như vậy không đủ để $f(x) \ge 0$ đâu.

Giáo viên môn Toán tại website : http://vted.vn





0 người đang xem chủ đề

0 thành viên, 0 khách, 0 thành viên ẩn danh